14

Prove the following

$$\int^1_0 \frac{\operatorname{Li}_2^2(x)}{x}\, dx = -3\zeta(5)+\pi^2 \frac{\zeta(3)}{3}$$

where

$$\operatorname{Li}^2_2(x) =\left(\int^x_0 \frac{\log(1-t)}{t}\,dt \right)^2$$

Ali Shadhar
  • 25,498
Zaid Alyafeai
  • 14,343

2 Answers2

14

Integrating once by parts, we find \begin{align} I=\int_0^{1}\frac{\mathrm{Li}_2^2(x)\,dx}{x}&=\int_0^{1}\mathrm{Li}_2(x)\,d\left(\mathrm{Li}_3(x)\right)=\\ &=\mathrm{Li}_2(1)\mathrm{Li}_3(1)+\int_0^{1}\frac{\mathrm{Li}_3(x)\,\ln(1-x)\,dx}{x}=\\ &=\zeta(2)\zeta(3)-\sum_{n=1}^{\infty}\sum_{k=1}^{\infty}\frac{1}{n^3k(n+k)},\tag{1} \end{align} where the last line is obtained by expanding $\mathrm{Li}_3(x)$ and $\ln(1-x)$ into series and integrating. Now if we denote $H_n=\sum_{k=1}^n\frac{1}{k}$ the $n$th harmonic number, the sum with respect to $k$ in (1) can be written as $$\sum_{k=1}^{\infty}\frac{1}{k(n+k)}=\frac{1}{n}\sum_{k=1}^{\infty}\left(\frac{1}{k}-\frac{1}{n+k}\right)=\frac{H_n}{n},$$ so that $$I=\zeta(2)\zeta(3)-\sum_{n=1}^{\infty}\frac{H_n}{n^4}.\tag{2}$$ Now using that $\zeta(2)=\frac{\pi^2}{6}$ and the formula (20) here (it is a particular case of a more general Euler sum (24)), namely, $$ \sum_{n=1}^{\infty}\frac{H_n}{n^4} = 3\zeta(5)-\frac{\pi^2}{6}\zeta(3),$$ we finally obtain the desired result for $I$. $\blacksquare$

Start wearing purple
  • 53,234
  • 13
  • 164
  • 223
  • 4
    Interestingly I proved that

    $$\int^1_0 \frac{\operatorname{Li}2^2(x)}{x},dx= \frac{2}{3},\sum{n\geq 1}\frac{H_n^{(2)}}{n^3}$$

    – Zaid Alyafeai Aug 09 '13 at 00:03
3

$$I=\int_0^1\frac{\operatorname{Li}_2^2(x)}{x}\ dx=\sum_{n=1}^\infty\frac1{n^2}\int_0^1x^{n-1}\operatorname{Li}_2(x)\ dx\overset{IBP}{=}\sum_{n=1}^\infty\frac1{n^2}\left(\frac{\zeta(2)}{n}-\frac{H_n}{n^2}\right)$$

$$=\zeta(2)\zeta(3)-(3\zeta(5)-\zeta(2)\zeta(3))=\boxed{2\zeta(2)\zeta(3)-3\zeta(5)}$$

Ali Shadhar
  • 25,498
  • How did you conclude that $$\sum_{n=1}^\infty \frac{1}{n^2} \left( \frac{\zeta(2)}{n} - \frac{H_n}{n^2} \right) = 2\zeta(2)\zeta(3) - 3\zeta(5) \hspace{2mm}?$$ I was trying to find closed form of $$\sum_{n=1}^\infty \frac{H_n}{n^4}$$ but I got the integral which is asked and while trying to solve that integral I am stuck in this sum. It is like a loop for me.... – Lucky Chouhan Aug 21 '23 at 04:31
  • 1
    @LuckyChouhan check this post https://math.stackexchange.com/q/469023/432085 and I agree it can be a loop. It depends on the derivation of Euler sum – Ali Shadhar Aug 21 '23 at 04:39
  • Thank you!! It is out of context, but I really loved 2nd edition of your book. – Lucky Chouhan Aug 21 '23 at 04:44
  • 1
    @LuckyChouhan thank you for the kind words and glad you like it – Ali Shadhar Aug 21 '23 at 06:22